exercise

69

Question 1.150

Calculate the dot product of \({\bf x}=(1,-1,0,2)\in {\mathbb R}^4\) and \({\bf y}=(1,2,3,4)\in{\mathbb R}^4\).

Question 1.151

In \({\mathbb R}^n\) show that

  • (a) \(2 \|{\bf x} \|^2 + 2 \|{\bf y}\|^2 = \| {\bf x} + {\bf y}\|^2 + \|{\bf x} - {\bf y} \|^2\) (This is known as the parallelogram law.)
  • (b) \(\|{\bf x} - {\bf y} \|\| {\bf x} + {\bf y}\|\le \|{\bf x} \|^2 + \|{\bf y} \|^2\)
  • (c) \(4 \langle {\bf x}, {\bf y}\rangle = \|{\bf x} + {\bf y} \|^2 - \|{\bf x} - {\bf y}\|^2\) (This is called the polarization identity.)

Interpret these results geometrically in terms of the parallelogram formed by x and y.

Verify the Cauchy–Schwarz inequality and the triangle inequality for the vectors in Exercises 3 to 6.

Question 1.152

\({\bf x} = (2,0,-1), {\bf y} = (4,0,-2)\)

Question 1.153

\({\bf x} = (1,0,2,6), {\bf y} = (3,8,4,1)\)

Question 1.154

\({\bf x} = (1,-1,1, -1,1), {\bf y} =(3,0,0,0, 2)\)

Question 1.155

\({\bf x}=(1,0,0,1),{\bf y}=(-1,0,0,1)\)

Question 1.156

Let \(\textbf{v}, \textbf{w} \in \mathbb{R}^n\). If \( \| \textbf{v} \| = \| \textbf{w} \|\), show that \(\textbf{v} + \textbf{w}\) and \(\textbf{v} - \textbf{w}\) are orthogonal.

Question 1.157

Suppose \(T\) is a triangle formed by placing three points on a circle, two of which lie on the circle’s diameter. Use the previous problem to show \(T\) is a right triangle.

Question 1.158

Compute \(AB, \hbox{det } A, \hbox{det } B, \hbox{det } (A B)\), and \(\hbox{det } (A+B)\) for \[ A = \Bigg[\begin{array}{@{}c@{\quad}r@{\quad}c@{}} 1 & -1 & 0 \\ 0 & 3 & 2 \\ 3 & 1 & 1 \end{array} \Bigg] \qquad \hbox{and } \qquad B = \Bigg[\begin{array}{@{}r@{\quad}c@{\quad}r@{}} -2 & 0 & 2 \\ -1 & 1 & -1 \\ 1 & 4 & 3 \end{array} \Bigg]. \]

Question 1.159

Compute \(AB, \hbox{det } A, \hbox{det } B, \hbox{det } (AB)\), and \(\hbox{det } (A+B)\) for \[ A = \Bigg[\begin{array}{@{}c@{\quad}c@{\quad}r@{}} 3 & 0 & 1\\ 1 & 2 & -1 \\ 1 & 0 & 1 \end{array} \Bigg] \qquad \hbox{and } \qquad B = \Bigg[\begin{array}{@{}c@{\quad}c@{\quad}r@{}} 1 & 0 & -1 \\ 2 & 0 & 1 \\ 0 & 1 & 0 \end{array} \Bigg]. \]

Question 1.160

Determine which of the following matrices are invertible: \[ A=\left[ \begin{array}{ccc} 1 & 2 & 3 \\ 0 & 1 & 1 \\ 0 & 3 & 3 \\ \end{array} \right] \quad B=\left[ \begin{array}{ccc} 0 & 0 & 3 \\ -1 & 1 & 19 \\ 2 & 3 & \pi \\ \end{array} \right] \quad C=\left[ \begin{array}{cc} 1 & 1 \\ 1 & 1 \\ \end{array} \right] \]

Question 1.161

For matrix \(A\) in the previous problem, find a nonzero \(\textbf{x} \in \mathbb{R}^3\) such that \(A\textbf{x}=\textbf{0}\).

Question 1.162

Use induction on \(k\) to prove that if \({\bf x}_1, \ldots , {\bf x}_k \in {\mathbb R}^n\), then \[ \|{\bf x}_1 + \cdots + {\bf x}_k\| \le \|{\bf x}_1 \| + \cdots + \|{\bf x}_k\|. \]

Question 1.163

Using algebra, prove the identity of Lagrange: For real numbers \(x_1, \ldots , x_n\) and \(y_1, \ldots , y_n.\) \[ \bigg( \sum_{i=1}^n x_i y_i \bigg)^2\! = \bigg( \sum_{i=1}^n x_i^2 \bigg) \bigg( \sum_{i=1}^n y_i^2 \bigg) {-} \sum_{i < j} (x_i y_j - x_j y_i)^2. \]

Use this to give another proof of the Cauchy–Schwarz inequality in \({\mathbb R}^n\),

Question 1.164

Prove that if \(A\) is an \(n \times n \) matrix, then

  • (a) \(\hbox{det } ( \lambda A) = \lambda^n \hbox{ det } A;\) and
  • (b) if \(B\) is a matrix obtained from \(A\) by multiplying any row or column by a scalar \(\lambda\), then \(\hbox{det } B = \lambda \hbox{ det } A\).

In Exercises 16 to 18, \(A\),\(B\), and C denote \(n \times n\) matrices.

Question 1.165

Is \(\hbox{det } (A+B) = \hbox{det } A + \hbox{det } B\)? Give a proof or counterexample.

Question 1.166

Does \((A+B) (A-B) = A^2 - B^2\)?

Question 1.167

Assuming the law \(\hbox{det } (AB) = ( \hbox{det } A)(\hbox{det } B)\), prove that det \((ABC) = ( \hbox{det } A) ( \hbox{det } B)(\hbox{det } C)\).

Question 1.168

(This exercise assumes a knowledge of integration of continuous functions of one variable.) Note that the proof of the Cauchy–Schwarz inequality (Theorem 4) depends only on the properties of the inner product listed in Theorem 1. Use this observation to establish the following inequality for continuous functions \(f,g \colon [0,1] \to {\mathbb R}\): \[ \bigg| \int_0^1 f (x) g (x)\ \ dx \bigg| \le \sqrt{ \int^1_0 [f (x) ]^2\ \ dx} \ \sqrt{\int_0^1[ g (x)]^2\ \ dx}. \]

Do this by

  • (a) verifying that the space of continuous functions from [0, 1] to \({\mathbb R}\) forms a vector space; that is, we may think of functions \(f,g\) abstractly as “vectors” that can be added to each other and multiplied by scalars.
  • (b) introducing the inner product of functions \[ f \,{\cdot}\, g = \int_0^1 f (x) g (x) \ dx \] and verifying that it satisfies conditions (i) to (iv) of Theorem 3.

Question 1.169

70

Define the transpose \(A^T\) of an \(n \times n\) matrix \(A\) as follows: the \(ij\)th element of \(A^T\) is \(a_{ji}\) where \(a_{ij}\) is the \(ij\)th entry of \(A\). Show that \(A^T\) is characterized by the following property: For all \({\bf x,y}\) in \({\mathbb R}^n\), \[ (A^T {\bf x}) \,{\cdot}\, {\bf y} = {\bf x} \,{\cdot}\, ( A {\bf y}). \]

Question 1.170

Verify that the inverse of \[ \Big[\begin{array} &a & b \\ c & d \end{array} \Big] \qquad \hbox{is} \qquad \frac{1}{ad- bc} \Big[\begin{array}{@{}r@{\quad}r@{}} d & -b \\ -c & a \end{array} \Big]. \]

Question 1.171

Use your answer in Exercise 21 to show that the solution of the system \begin{eqnarray*} && ax+ by = e \\ && cx + dy =f\\[-21.5pt] \end{eqnarray*} is \[ \Big[\begin{array}{c} x \\ y \end{array} \Big] = \frac{1}{ad-bc} \Big[\begin{array}{@{}r@{\quad}r@{}} d & -b \\ -c & a \end{array} \Big] \Big[\begin{array}{c} e \\ f \end{array} \Big]. \]

Question 1.172

Assuming the law \(\hbox{det } (AB) = ( \hbox{det } A) ( \hbox{det } B)\), verify that \((\hbox{det } A) (\hbox{det } A^{-1})=1\) and conclude that if \(A\) has an inverse, then \(\hbox{det } A \ne 0\).

Question 1.173

Find two \(2 \times 2\) matrices \(A\) and \(B\) such that \(AB=0\) but \(BA \neq 0\).